What is the value of r in the equation −4/3r − 1 = −2 ?

Answers

Answer 1

Answer:

r = 3/4

Step-by-step explanation:


Related Questions

write an algebraic expression for the word expression.
the sum of 9 and a number x​

Answers

Answer:

9 + x

Step-by-step explanation:

i think this is what you meant

Answer:

Write each phrase as an algebraic expression. In Example 2, each algebraic expression consisted of one number, one operation and one variable.

...

Algebraic Expressions With Example Problems and Interactive Exercises.

Phrase Expression

the sum of nine and a number x 9 + x

Step-by-step explanation:

Hopes this helps\

Mark as brainliest

Trevor simplifies a linear equation

Answers

Answer:

C :3

Step-by-step explanation:

Whenever our simplification of a linear equation ends in a true statement, in this case 6=6, then our linear equation has infinitely many solutions.

Therefore, our answer is C.

even if we don’t know what x is we’ll still get an infinite amount of solutions

HOPE THIS HELPED :DDDD

4++++++++++++++++++++++++++++++++=4=

Answers

Answer:

is equal to 0

Step-by-step explanation:

points Lang habol

A= 3x3x3x3x3x5x7x7x7
B= 2x2x2x3x7x7x7x7
Find the Lowest Common Multiple (LCM) of A and B.
Please help

Answers

Answer:

LCM(57624, 416745) = 23,337,720 or  LCM = 23337720

Step-by-step explanation:

List all prime factors for each number.

Prime Factorization of 57624 is:

2 x 2 x 2 x 3 x 7 x 7 x 7 x 7  =>  23 x 31 x 74

Prime Factorization of 416745 is:

3 x 3 x 3 x 3 x 3 x 5 x 7 x 7 x 7  =>  35 x 51 x 73

For each prime factor, find where it occurs most often as a factor and write it that many times in a new list.

The new superset list is

2, 2, 2, 3, 3, 3, 3, 3, 5, 7, 7, 7, 7

Multiply these factors together to find the LCM.

LCM = 2 x 2 x 2 x 3 x 3 x 3 x 3 x 3 x 5 x 7 x 7 x 7 x 7 = 23337720

In exponential form:

LCM = 23 x 35 x 51 x 74 = 23337720

LCM = 23337720

Therefore,

LCM(57624, 416745) = 23,337,720

What is the best estimate for this sum?

1/8+1/6


Options here:

The sum will be close to 1/4

The sum will be close to 1/2

The sum will be close to 1


*Disclaimer* show work

Answers

It's 7/24 so about 0.292
therefore it will be close to 1/4

or you calculate 1/8 = 0.125 and 1/6 = 0.1666 adds up close to 1/4

Sally saved 65% of her pay check. If she made $500, how much did she save?

Answers

Answer: 325 dollars

Step-by-step explanation:

In order to find out a percentage of somethin, you can simply multiply and divide

[tex]500*65=32500[/tex]

32500÷100= 325

What is the probability for a bowler to score over 136? The mean for the set is 130, and the standard deviation is 3. So I know that it is Mean + 2(Standard Deviation), so is it 2.35% or 2.5%?

Answers

Answer:

135%

Step-by-step explanation:

Draw a diagram of the archway modeled by the equation y = x^2 + 5x + 24. Fine and label the y-intercept and the x-intercepts on the sketch. Then find and label the width of the archway at its base and the height of the archway at its highest point, assuming the base of the archway is along the x-axis ​

Answers

Answer:

1) Please find the attached drawing of the archway created with MS Excel

2) The y-intercept is (0, 24)

The x-intercepts are (-3, 0), and (8, 0)

3) The width of the archway at its base is 11

The height of the archway at its highest point = 30.25

Step-by-step explanation:

1) Please find the attached drawing of the archway created with MS Excel

2) The y-intercept is (0, 24)

The x-intercepts are (-3, 0), and (8, 0)

3) The width of the archway at its base is 11

The height of the archway at its highest point = 30.25

Step-by-step explanation:

The given function representing the archway is y = -x² + 5·x + 24

1) Please find attached the required drawing of the archway created with MS Excel

2) The y-intercept is given by the point where x = 0

Therefore, we have, the y-value at the y-intercept = -0² + 5×0 + 24 = 24

The y-intercept = (0, 24)

The x-intercept is given by the point where y = 0

Therefore, the x-values at the x-intercept are found using the following equation;

0 = -x² + 5·x + 24

x² - 5·x - 24 = 0

By inspection, we have;

x² - 8·x + 3·x - 24 = 0

x·(x - 8) + 3·(x - 8) = 0

∴ (x + 3) × (x - 8) = 0

Either (x + 3) = 0, and x = -3, or (x - 8) = 0, and x = 8

Therefore, the x-intercepts are (-3, 0), and (8, 0)

3) The width of the archway at its base = The distance between the x-values at the two x-intercepts

∴ The width of the archway at its base = 8 - (-3) = 11

The highest point of the arch is given by the vertex of the parabola, y = a·x² + b·x + c, which has the x-value of the vertex = -b/(2·a)

∴ The x-value of the vertex of the given parabola, y = -x² + 5·x + 24, is x = -5/(2×(-1)) = 2.5

Therefore;

The y-value of the vertex, is y = -(2.5)² + 5×2.5 + 24 = 30.25 = The height of the archway at its highest point

∴ The height of the archway at its highest point = 30.25

The vertex of the parabola is at (-2.5, 17.75). And the y-intercept of the parabola is at (0, 24).

What is the equation of the parabola?

Let the point (h, k) be the vertex of the parabola and a be the leading coefficient.

Then the equation of the parabola will be given as,

y = a(x - h)² + k

The equation of the parabola is given as,

y = x² + 5x + 24

Convert the equation into a vertex form, then we have

y = x² + 5x + 25/4 - 25/4 + 24

y = (x + 5/2)² + 71/4

y = (x + 2.5)² + 17.75

The vertex of the parabola is at (-2.5, 17.75). The y-intercept of the parabola is given as,

y = (0)² + 5(0) + 24

y = 24

The y-intercept of the parabola is at (0, 24).

More about the equation of the parabola link is given below.

https://brainly.com/question/20333425

#SPJ2

Evaluate
3
2
+
(

k
)
+
(

2
)
2
3

+(−k)+(−2)start fraction, 3, divided by, 2, end fraction, plus, left parenthesis, minus, k, right parenthesis, plus, left parenthesis, minus, 2, right parenthesis where
k
=

5
2
k=−
2
5

k, equals, minus, start fraction, 5, divided by, 2, end fraction.

Answers

Answer:

Iam very confused on this Answer please help me  

Step-by-step explanation:


A television set costs $4500 cash. When bought on hire-purchase, a deposit of $450 is
required, followed by 12 monthly payments of $400. How much is saved by paying cash?

Answers

Answer:

$750

Step-by-step explanation:

did the problem

2 circles labeled Set A and Set B overlap. Set A contains 1, set B contains 3, and the overlap of the 2 circles contains 2. The number 4 is outside of the circles. On the Venn diagram, which region(s) represent the intersection of Set A and Set B (Aâ©B)? II I and III I, II, and III I, II, III, and IV.

Answers

The intersection of sets is the point that represents the common point of multiple sets.

The region that represents the intersection of sets A and B is region II.

From the question, we understand that sets A and B have a common set element at II.

This means that region II is the intersection of both sets.

Hence, the region that represents the intersection of sets A and B is II.

Read more about set intersection at:

https://brainly.com/question/6779867

Answer:

II is the answer

Step-by-step explanation:

Edge 2022

From the observation deck of a skyscraper, Jamal measures a 51° angle of depression
to a ship in the harbor below. If the observation deck is 1123 feet high, what is the
horizontal distance from the base of the skyscraper out to the ship? Round your
answer to the nearest hundredth of a foot if necessary.

Answers

Answer:

Your answer is 909.39 feet

Step-by-step explanation:

Your answer is 909.9

bryan has some 3 cent stamps and some 4 cent stamps. what is the least number of stamps he can combine so the value of the stamps is 33 cents?

Answers

Answer:

The least number of stamps required is [tex]9[/tex]

Step-by-step explanation:

Let the number of [tex]3[/tex] cent stamps be [tex]x[/tex] and [tex]4[/tex] cent stamps be [tex]y[/tex]

We have

[tex]3x+4y=33[/tex]

The minimum number is obtained when more  [tex]4[/tex] cent stamps are used

Here [tex]y[/tex] cannot be greater than [tex]8[/tex] since [tex]\frac{33}{4} <9[/tex]

Substitute [tex]y=8[/tex]

[tex]3x+4\times 8=33\\\\3x=1\\\\x=\frac{1}{3}[/tex]

Not possible since [tex]x[/tex] is not a fraction

Substitute [tex]y=7[/tex]

[tex]3x+4\times 7=33\\\\3x=5\\\\x=\frac{5}{3}[/tex]

Not possible since [tex]x[/tex] is not a fraction

Substitute [tex]y=6[/tex]

[tex]3x+4\times 6=33\\\\3x=9\\\\x=\frac{9}{3}\\\\=3[/tex]

Possible

Hence minimum number of stamps is

[tex]=x+y\\\\=3+6\\\\=9[/tex]

y^2 - 19 = 0 find the sum and product

Answers

Answer: Sum of the roots:  √19 - √19 = 0

Product of roots:  (√19)(-√19) = -19

Step-by-step explanation:

y2 - 19 = 0

add 19 to both sides

y2 = 19

take square root of both sides

y = ±√19

So y = √19 or y = -√19

Write an expression for the volume of the figure as a polynomial in standard form. (multiple choice)

Answers

Answer:

3.

2x^3+10x^2+14x+6

Step-by-step explanation:

according to figure,

v=w•l•h

(2x+2)(x+1)(x+3)

=(2x+2)x(x+1)+3(X+1)

=(2x+2)(x^2+4x+3)

=2x(x^2+4x+3)+2(x^2+4x+3)

=2x^3+10x^2+14x+6

The volume of the figure as a polynomial in standard form is

V = 2x³ + 10x² + 14x + 6.

What is a cuboid?

A closed three-dimensional geometric shape called a cuboid is surrounded by six rectangular plane sections.

The given figure is a cuboid with dimensions (2x + 2), (x + 1), and (x + 3).

The volume of the cuboid is the multiplication of its dimension, therefore, the volume of the cuboid is:

V = (2x +2)(x + 1)(x + 3)

V = (2x² + 2x +2x + 2)(x + 3)

V = (2x² + 4x + 2)(x + 3)

V = 2x³ + 6x² + 4x² + 12x + 2x + 6

V = 2x³ + 10x² + 14x + 6

Hence, the volume of the figure as a polynomial in standard form is

V = 2x³ + 10x² + 14x + 6.

Learn more about the cuboids:

https://brainly.com/question/29424737

#SPJ2

Mike wants to fence in part of his backyard. He wants the length of the fenced-in area to be at least 20 feet long, l ≥ 20. He has 200 feet of fencing. The inequality that models the possible perimeter of the yard is 2l 2w ≤ 200. Which are possible dimensions for Mike’s backyard? Check all that apply. W = 50 ft; l = 10 ft w = 10 ft; l = 50 ft w = 20 ft; l = 60 ft w = 90 ft; l = 30 ft w = 50 ft; l = 40 ft.

Answers

The inequality that models the possible perimeter of the yard is 2(l + w) ≥ 200

The possible dimension of the width is 90 feet

The formula for calculating the perimeter of the fence is expressed as:

P = 2(l + w)

l is the length of the fencew is the width of the fence

If he has 200 feet of fencing, then:

2(l + w) ≥ 200

If the length of the fenced-in area to be at least 20 feet long, the equation becomes:

2(20 +w) ≥ 200

40 + 2w ≥ 200

2w ≥  200 - 40

2w≥ 160

w≥  80

Hence the possible dimension of the width is 90 feet

Learn more on inequalities here: https://brainly.com/question/10709615

find the volume of this figure:

Answers

Answer:

= 5l. 66 in^3

Step-by-step explanation:

Area of cross section (A) = 1/2×b×h

=1/2 ×3.5×4.1

=7.175

Volume =AXH

= 7.175× 7.2

= 5l. 66 in^3

X
18. MODELING WITH MATHEMATICS The inequality
3x + 2y 2 93 represents the numbers x of multiple-
choice questions and the numbers y of matching
questions you can answer correctly to receive an A on
a test. You answer 20 multiple-choice questions and
18 matching questions correctly. Do you receive an A
on the test? Explain.

Answers

Answer:

  yes

Step-by-step explanation:

Put the numbers into the inequality and see if it is true.

Given x = 20, y = 18, ...

  3x + 2y = 3(20) +2(18) = 60 +36 = 96 ≥ 93 . . . . true

You have met the requirements for an A on the test.

For f(x) = -1/5x - 14 find x when f (x) = -8

Answers

Hi!

30 = x

f(x) = -1/5x - 14 find x when f (x) = -8

Set the equation equal to -8

-8 = -1/5x - 14

Add 14 to both sides.

6 = -1/5x

Multiply both sides by -5.

30 = x

Hope this helps! :D

Help me solve for x. :>

Answers

Answer:

x = 9

Step-by-step explanation:

3 ㏒₄x  = 2 ㏒₄27

㏒₄ x³ = ㏒₄ 27²

As ㏒₄ = ㏒₄

You can write this as,

x³ = 27²

x ³ = 27 × 27

x³ = 3³ × 3³

x ³ = 3⁶

x ³ = 729

x ³ = 9³

There fore,

x = 9

Hope this helps you :-)

Solve this question with Statements and reasoning.

Answers

Here

[tex]\\ \sf\longmapsto PD\cong PC[/tex]

As

P is the midpoint of AC and BD

[tex]\\ \sf\longmapsto AC=BD[/tex]

[tex]\\ \sf\longmapsto AP+PC=PD+BP[/tex]

Put PD=PC

[tex]\\ \sf\longmapsto AP+PC=PC+PB[/tex]

Cancel PC

[tex]\\ \sf\longmapsto AP=BP[/tex](Proved)

simplify: 2x^2+15x-30

A= 2x^2
B= 15x
C= -30​

Answers

(x-5)(x-6)

I dunno, its my question, just answering it for fun

Please help me with my math problem!! NO LINKS PLEASE!! IT IS DUE TONIGHT AT 11:59PM!! WILL GIVE BRAINLIEST!! :)

Answers

Step-by-step explanation:

Hi, i think this is how to do it. I'm sorry if I do it wrong!

(I need these answered fast and with work and explanation)
A)What is the conditional probability of being on the marching band, given that you know
the student plays a team sport? Show your work.

b. What is the probability of being on the marching band, and how is this different from part
(a)? Explain completely.

C.
Are the two events, {on the marching band) and {on a team sport} associated? Use
probabilities to explain why or why not.

Answers

Answer:

most I know I don't know and I don't want to know

when using the zero product rule you know at least one of the factors must equal *blank*

Answers

Answer:

The Principle of Zero Products states that if the product of two numbers is 0, then at least one of the factors is 0.

Step-by-step explanation:

Answer:

when using the zero product rule you know at least one of the factors must equal 0 because and most places anything that is put with zero equal zero because zero have no value

Step-by-step explanation:

Employee: "We have a promotion that will allow us to beat our competitors' prices and save you money. Did you notice that our competitor is selling that same item for $150.00?
Customer: "Yes"
Employee: "Our price for that item is $120.00, for a savings of __________."10% 15% 20% 30% 40%

Answers

Answer:

20%

Step-by-step explanation:

120 / 150 = 0.8 or 80%, so the difference is 20%

The discount percentage is 20%.

Given that the competitor is selling an item for $ 150.00, while the price for that item in the store is $ 120.00, the following calculation must be performed to determine the discount percentage:

150 = 100 120 = X 120 x 100/150 = X 80 = X 100 - 80 = X 20 = X

Therefore, the discount percentage is 20%.

Learn more about maths in https://brainly.com/question/26075432

((7,4),(6,3),(5,2)
Function?
Domain:
Range:

Answers

Answer:

see explanation

Step-by-step explanation:

For a function each value of x in the domain must map to exactly one unique value of y in the range.

This is the case here so it is a function.

The domain is the x- coordinates of the points

domain is { 5, 6, 7 }

The range is the y- coordinates of the points

range is { 2, 3, 4 }

PLEASE HELP!!!!
ILL MARK AS BRAINIEST!!!!
ANSWER HOW MUCH U CAN!!!

Answers

Answer:

1) C

2) B

3) D

4) A

5) B

Step-by-step explanation:

Hope this helps

MAy I get braineist pls???????????

Answer:

MARK ME AS BRAINLIEST!!!

Step-by-step explanation:

1. A

2. B

3. D

4. A

5. B

HAVE A NICE DAY!!!

:)

The vertices of quadrilateral PQRS are listed.

P(3,7), Q(6,-2), R(0,-4), S(-3,5)

Which of the following is the strongest classification that identifies quadrilateral PQRS?

A.
Quadrilateral PQRS is a rectangle.
B.
Quadrilateral PQRS is a trapezoid.
C.
Quadrilateral PQRS is a square.
D.
Quadrilateral PQRS is a parallelogram.

Answers

Answer:

Rectangle

Step-by-step explanation:

Did it on

Option A is correct.

As, Quadrilateral PQRS is a rectangle because it has four right angles.

What is a rectangle?

A rectangle is a quadrilateral with four right angles. It can also be defined as: an equiangular quadrilateral, since equiangular means that all of its angles are equal; or a parallelogram containing a right angle. A rectangle with four sides of equal length is a square.

here, we have,

Explanation:

Given:

The coordinate of vertices of quadrilateral PQRS are P(3,7), Q(6,-2), R(0,-4), S(-3,5).

Slope for two points (x1,y1) and (x2,y2) is given by:

m = y1-y2/ x1 - x2

.so, we get,

The slope of PQ is = -9/3 = -3

The slope of QR is = -2/-6 = 1/3

The slope of RS is = -3

The slope of SP is = 1/3

we know that,

The slopes of perpendicular lines are opposite reciprocals.

we get,

∴ all pair of sides has slopes that are negative reciprocals;

this means the figure  has 4 right angle, so it is  a rectangle.

Therefore, Quadrilateral PQRS is a rectangle because it has four right angles.

To learn more on rectangle click:

brainly.com/question/29123947

#SPJ2

Which equation represents the vertical line passing through (14,-16)?.

Answers

The vertical line shall have its x-coordinate equal to 14 because the points (14, -16) is on this vertical line.

Given the following data:

Points (x, y) = (14, -16)

To find an equation of the vertical line that passes through the given points (14, -16):

The standard form of an equation of line is given by the formula;

[tex]y=mx+b[/tex]

Where:

x and y are the points.m is the slope.b is the intercept.

For any equation of a straight line, the x-coordinate of a vertical line will always be the same irrespective of the point chosen on the vertical line. because a vertical line is perpendicular to the x-axis.

Deductively, the vertical line shall have its x-coordinate equal to 14 because the points (14, -16) is on this vertical line.

Read more: https://brainly.com/question/2707920

Other Questions
23. Where are earthquakes most likely tooccur?A where Earth's tectonic plates aremoving togetherB wheremagma cools slowly beneathEarth's surfaceC where Earth's lithosphere is inconstant, slow motionD where Earth's seafloor is spreadinginto deep ocean trenches Khalid dissolves lemon juice in water. He explains to his friend that both lemon juice and water are solvents and there is no solute in this solution. Plsssssssssss helppppppp Solve for x.X/-6 =-7 what identifies a cell or a range of cells on a worksheet, and tells excel where to look for values or data you to use in a formula A 67-kilogram cannon fires a 1 kilogram cannonball at 45 m/sec. If the cannon is on wheels, at what speed does it move backward? Which scenario describes an interaction between two of Earths spheres?Water flows from a stream to a lake.Gravity moves rocks to another location.Lions use energy to catch other animals for food.Bears dig big holes in the ground to protect their young. MERRY CHRISTMAS EVERYONE I HOPE YOU GUYS HAVING A WONDERFUL CHRISTMAS DAY AND CHRISTMAS EVENING!HERE'S A 50 POINTS FREE!THIS LAST ONE! 9.aWhat is the energy of a wave having a frequency of 5.71 x 1013 Hz? why does the u.s government require men to register with the selective service ? A rug in the classroom is 3/4 yard wide and 8/6 yards long. What is the area of the rug in square yards?PLEASE BE ANSWERED IN 30 MINS TOPS Which sentence is the correct punctuation usage of the semicolon, colon, or comma?AProfessor Brown has left; the laboratory, however, you may still be able to reach her through email.BProfessor Brown; has left the laboratory however; you may still be able to reach her through email.CProfessor Brown has left the laboratory, however you may; still be able to reach her through email.DProfessor Brown has left the laboratory; however, you may still be able to reach her through email. Here is fs fs da last graph I need help so someone plz help me Which statement best reflects the purpose of the passage?A. Congress and the states can work to amend the Constitution,but all citizens have the right to vote directly to passamendments.B. Amending the Constitution is a simple process that should beconsidered often.O C. The Constitution cannot be changed, but the states can callfor a new constitution to be written.D. The Constitution can be changed if those changes havewidespread support in Congress and the states.SUBMIT PLEASE HELP FAST! DUE TODAY AND CANT HAVE A 0! Computers were originally developed to accomplish various tasks in relative isolation, very different from the collaboration and communication we see today.A. TrueB. False 3(x+2)+5x how do i simplify that expression? A lifeguard received 15 hours of first aid training and 10 hours of CPR training. What is the ratio of hours of CPR training to hours of first aid training? could someone help me with this please, worth 20 points The extended warranty on a $960 dishwasher is 21% of the purchase price and lasts for eight years. What is the effective cost per year of the extended warranty? a. $28. 80 b. $45. 20 c. $23. 78 d. $25. 20.